LSAT and Law School Admissions Forum

Get expert LSAT preparation and law school admissions advice from PowerScore Test Preparation.

User avatar
 Dave Killoran
PowerScore Staff
  • PowerScore Staff
  • Posts: 5853
  • Joined: Mar 25, 2011
|
#27043
Complete Question Explanation
(The complete setup for this game can be found here: lsat/viewtopic.php?t=11335)

The correct answer choice is (C)


The combination of the rules and inferences allows you to eliminate each of the incorrect answers in this Could Be True problem:

Answer choice (A): F and H can never be assigned to the same panel because from the first rule F and G must be assigned to the same panel, and from the fourth rule G cannot be assigned to the same panel as H.

Answer choice (B): G and P can never be assigned to the same panel because from the first rule F and G must be assigned to the same panel, and from the third rule F cannot be assigned to the same panel as P.

Answer choice (D): F and M can never be assigned to the same panel because each is a part of a different block, and there is not enough room in any of the groups to accommodate both blocks.

Answer choice (E): J and M can never be assigned to the same panel because from the second rule K and M must be assigned to the same panel, and from the fifth rule M cannot be assigned to the same panel as J.

Only the two variables in answer choice (C) can be assigned to the same panel as each other (although that panel cannot be the Oceans panel), and thus answer choice (C) is correct.

Get the most out of your LSAT Prep Plus subscription.

Analyze and track your performance with our Testing and Analytics Package.